Problema di cauchy

Messaggioda StrilingAlQuadrato » 11/02/2024, 11:47

Ciao a tutti :)

$ { ( y'=(y^2-2y)/(y-1)*sin^2t ),( y(pi) = 3 ):} $

In breve:

$ (y-1)/(y^2-2y) dy= sin^2t dt $

integrando a entrambi i membri

$ ln|y^2-2y|=t - sin(2t)/2 + c $

$ |y(y-2)| = k * e^((2t - sin(2t))/2) $

se sostituisco
$ y(pi) = 3 $
$ |3(3-2)| = k * e^((2*pi - sin(2*pi))/2) $
$ 3 = k * e^(pi) $
$ k = 3/e^(pi) $

Poi non so cosa fare, in quanto non so come isolare la y.. :? Grazie in anticipo
Ultima modifica di StrilingAlQuadrato il 11/02/2024, 15:56, modificato 2 volte in totale.
StrilingAlQuadrato
Starting Member
Starting Member
 
Messaggio: 40 di 46
Iscritto il: 30/10/2022, 17:47

Re: Problema di cauchy

Messaggioda Quinzio » 11/02/2024, 15:21

$ |y(y-2)| = k * e^((2t - sin(2t))/2) $

Puoi partire da $t= \pi, y = 3$, quindi togliere il modulo, siccome $y(y-2) > 0$ e osservare cosa fa la derivata.
Vedi che la derivata e' positiva, quindi la funzione cresce, quindi $y(y-2) > 0$, e quindi a destra di $t = \pi$ togliamo il modulo e ce ne dimentichiamo.
Per esplicitare la $y$ devi risolvere l'equazione quadratica

$ y^2 -2y - k * e^((2t - sin(2t))/2) = 0 $

come una normale equazione $ay^2+by+c = 0$.

Ovviamente devi fare attenzione ai segni, alle due soluzioni.

Andando indietro nel tempo si vede che per $y= 2$ la derivata si annulla, quindi la $y$ arriva a $2$ e poi rimane fissa su quel valore.
Quinzio
Cannot live without
Cannot live without
 
Messaggio: 5836 di 10548
Iscritto il: 24/08/2010, 06:50

Re: Problema di cauchy

Messaggioda StrilingAlQuadrato » 11/02/2024, 16:11

Grazie per la risposta, ma onestamente non ho capito molto. :?

$ y_(1,2) = 2 +- 2sqrt(1+k * e ^((2t-sin2t)/2)) $

Non so se devo lasciare così.. ma se sostuisco $ k=3/e^pi $ e $ t = pi $

$ y_(1,2) = 2 +- 2sqrt(1+3/e^pi * e ^((2pi-sin2pi)/2)) = 2 +- 2sqrt(4) = 2 +- 4 $
$ y_1 = -2 $ e $ y_2 = 6 $
StrilingAlQuadrato
Starting Member
Starting Member
 
Messaggio: 42 di 46
Iscritto il: 30/10/2022, 17:47

Re: Problema di cauchy

Messaggioda Quinzio » 11/02/2024, 17:37

Pero' le basi non vanno dimenticate... la soluzione e'

$ y_(1,2) = 1 +- sqrt(1+k * e ^((2t-sin2t)/2)) $

Inoltre mi sembra che ci sia un altro errore in $ ln|y^2-2y|=t - sin(2t)/2 + c $

Non hai dimenticato $1/2$ ?

$ 1/2 ln|y^2-2y|=t - sin(2t)/2 + c $
Quinzio
Cannot live without
Cannot live without
 
Messaggio: 5838 di 10548
Iscritto il: 24/08/2010, 06:50

Re: Problema di cauchy

Messaggioda StrilingAlQuadrato » 11/02/2024, 18:41

Ops.. ho dimenticato il diviso 2.

$ y_(1,2) = 1 +- sqrt(1+k * e ^((2t-sin2t)/2)) $
quindi $ y_(1) = -1 $ e $ y_(2) = 3 $.


Ho ricontrollato, se non ho sbagliato sarebbe:
$ 1/2*ln|y^2-2y|= 1/2t - sin(2t)/4 + c $
e ho moltiplicato tutto per 2, quindi diventa:
$ ln|y^2-2y|= t - sin(2t)/2 + c $
StrilingAlQuadrato
Starting Member
Starting Member
 
Messaggio: 43 di 46
Iscritto il: 30/10/2022, 17:47

Re: Problema di cauchy

Messaggioda Quinzio » 11/02/2024, 18:52

Si giusto, adesso dovrebbe essere tutto a posto.

Delle due soluzioni prendiamo $y_2$.
Quinzio
Cannot live without
Cannot live without
 
Messaggio: 5842 di 10548
Iscritto il: 24/08/2010, 06:50

Re: Problema di cauchy

Messaggioda StrilingAlQuadrato » 11/02/2024, 18:55

Esatto, grazie mille :)
StrilingAlQuadrato
Starting Member
Starting Member
 
Messaggio: 45 di 46
Iscritto il: 30/10/2022, 17:47

Re: Problema di cauchy

Messaggioda gugo82 » 12/02/2024, 17:33

StrilingAlQuadrato ha scritto:$ { ( y'=(y^2-2y)/(y-1)*sin^2t ),( y(pi) = 3 ):} $

L'equazione è del tipo a variabili separabili con secondo membro $f(t,y) = T(t) Y(y)$, in cui:

$T(t) := sin^2 t$ e $Y(y) := (y(y-2))/(y - 1)$,

definito in $Omega := RR xx (RR \setminus \{ 1\})$ ed ivi di classe $C^oo$.
Per notissimi fatti di teoria, il P.d.C. ha unica soluzione locale che si prolunga in un'unica soluzione massimale di classe $C^oo$, chiamiamola $y(t;pi, 3)$, il cui grafico non attraversa (per unicità locale) quello delle soluzioni costanti $y_***(t) = 0$ ed $y^*** (t)=2$ né la retta di equazione $y=1$ esclusa dal dominio del secondo membro. Dato che $y(pi;pi,3)=y(pi)=3 > 2=y^***(t)$, il grafico della soluzione vive tutto al disopra di quello di $y^***$, cioè della retta di equazione $y=2$. Conseguentemente, si ha $y(t;pi,3) > 2$ nell'intervallo $I = ]a,b[$ in cui è definita la soluzione massimale e, conseguentemente, anche $y^'(t;pi,3) > 0$ in $I$.
Dunque $y(t;pi,3)$ è strettamente crescente in $I$ ed ammette limiti negli estremi sinistro e destro di $I$, cioè esistono:

$lim_(t -> a^+) y(t;pi,3) = lambda$ e $lim_(t -> b^(-)) y(t;pi,3) = Lambda$.

Dato che $y(t;pi,3)>2$, si ha $lambda >= 2$; se $a$ fosse finito, per noti fatti di teoria, la soluzione massimale sarebbe prolungabile a sinistra di $a$, ma ciò è assurdo perché allora $y(t;pi,3)$ non sarebbe massimale; quindi $a=-oo$; il teorema dell'asintoto allora ci dà come unica possibilità $lambda = 2$, sicché $y(t;pi,3)$ ha come asintoto orizzontale a sinistra la retta di equazione $y=2$ (ossia il grafico di $y^***$).
Analogamente, visto che $Y(y)$ è un rapporto tra polinomi con numeratore di un grado maggiore del denominatore, è possibile trovare costanti $A, B >=0$ tali che:

$|Y(y)| <= A|y| + B$

cosicché:

$|f(t,y)| <= Asin^2 t |y| + Bsin^2 t <= A|y| + B$

in $Omega$; ne viene che $f$ è sublineare in $y$ (uniformemente rispetto a $t$) e ciò assicura che $b=+oo$.

Quindi possiamo già dire che la soluzione massimale è definita e $C^oo$ in tutto $R$, che è strettamente crescente e che ha un asintoto orizzontale a sinistra... Il tutto senza calcolare in alcun modo la soluzione!

A questo punto, possiamo anche provare a calcolarla esplicitamente, la soluzione.
Tenendo presente che $y(\pi) = 3$ e $y(t) > 2$, troviamo:
\[
\begin{split}
\frac{(y(t) - 1)\ y^\prime (t)}{y^2 (t) - 2y(t)} = \sin^2 t \quad &\Rightarrow \quad \int_\pi^t \frac{(y(\tau) - 1)\ y^\prime (\tau)}{y^2 (\tau) - 2y(\tau)}\ \text{d} \tau = \int_\pi^t \sin^2 \tau\ \text{d}\tau\\
&\stackrel{u = y(t)}{\Rightarrow} \quad \int_3^{y(t)} \frac{u - 1}{u^2 - 2u}\ \text{d} u = \int_\pi^t \sin^2 \tau\ \text{d}\tau\\
&\Rightarrow \quad \left. \frac{1}{2}\ \log (u^2 - 2u)\right|_3^{y(t)} = \frac{1}{2} (t - \sin t \cos t - \pi)\\
&\Rightarrow \quad \log \frac{y^2(t) - 2y(t)}{3} = \underbrace{t - \sin t \cos t - \pi}_{\Theta (t)} \\
&\Rightarrow \quad y^2(t) - 2y(t) - 3e^{\Theta(t)} = 0\; ;
\end{split}
\]
l'ultima equazione, che è di secondo grado in $y(t)$, ha discriminante ridotto:
\[
\frac{\Delta}{4} = 1 + 3e^{\Theta (t)} > 0
\]
quindi ha due soluzioni, delle quali una è positiva ed una è negativa per la regola di Cartesio; dato che la nostra soluzione è $>2$, dobbiamo prendere quella positiva, che è:
\[
y(t) = 1 + \sqrt{1 + 3e^{\Theta (t)}}\; .
\]
Pertanto:
\[
y(t;\pi, 3) = 1 + \sqrt{1 + 3e^{t - \sin t \cos t - \pi}}
\]
è la soluzione massimale del problema di Cauchy assegnato.
Sono sempre stato, e mi ritengo ancora un dilettante. Cioè una persona che si diletta, che cerca sempre di provare piacere e di regalare il piacere agli altri, che scopre ogni volta quello che fa come se fosse la prima volta. (Freak Antoni)
Avatar utente
gugo82
Cannot live without
Cannot live without
 
Messaggio: 26981 di 44972
Iscritto il: 12/10/2007, 23:58
Località: Napoli

Re: Problema di cauchy

Messaggioda Quinzio » 12/02/2024, 18:38

Un grazie a gugo per questo contributo di qualita'.
Quinzio
Cannot live without
Cannot live without
 
Messaggio: 5845 di 10548
Iscritto il: 24/08/2010, 06:50

Re: Problema di cauchy

Messaggioda gugo82 » 12/02/2024, 22:36

Quinzio ha scritto:Un grazie a gugo per questo contributo di qualita'.

Ancora ci difendiamo... :lol:

Grazie a te. :wink:
Sono sempre stato, e mi ritengo ancora un dilettante. Cioè una persona che si diletta, che cerca sempre di provare piacere e di regalare il piacere agli altri, che scopre ogni volta quello che fa come se fosse la prima volta. (Freak Antoni)
Avatar utente
gugo82
Cannot live without
Cannot live without
 
Messaggio: 26985 di 44972
Iscritto il: 12/10/2007, 23:58
Località: Napoli


Torna a Analisi matematica di base

Chi c’è in linea

Visitano il forum: Nessuno e 1 ospite